Plz help with this question! What is 1/3 * 3/5 * 5/7 ... * 97/99

Answers

Answer 1

Answer:

Step-by-step explanation:

(1/3)*(3/5)*(5/7)*.......*(93/95) * (95/97) * (97/99)

The denominator of each fraction will cancel out the numerator of the next fraction.

Take a close look

Therefore the result is 1/99

HOPE IT HELPS!!

Answer 2

The product of the geometric series  is 1/99.

The product is a geometric series with first term 1/3 and common ratio 2/3. The number of terms is 49. The sum of the series is 1/99.

Here's the explanation:

A geometric series is a series where each term is multiplied by a constant value to get the next term. The constant value is called the common ratio.

In this case, the first term is 1/3 and the common ratio is 2/3. This means that each term is multiplied by 2/3 to get the next term.

The number of terms in the series is 49. This means that there are 49 terms in the series, starting with 1/3 and ending with 97/99.

The sum of the series is 1/99. This can be found using the formula for the sum of a geometric series:

S = a × (1 - [tex]r^{n}[/tex]) / 1 - r

where a is the first term, r is the common ratio, and n is the number of terms.

In this case, a = 1/3, r = 2/3, and n = 49. Substituting these values into the formula, we get:

S = (1/3) × (1 - (2/3)49) / 1 - (2/3) = 1/99

Therefore, the product is 1/99.

Learn more about geometric series  here: brainly.com/question/30264021

#SPJ2


Related Questions

I NEED HELP PLEASE ! I GIVE 5 STARS !

Answers

Answer:

2

Step-by-step explanation:

Remember, a radical is the same thing as an exponent, but as a fraction. Therefore, instead of taking the 12th of the root, you can put [tex]8^4[/tex] to the 12th power like this: [tex](8^4)^{\frac{1}{12} }[/tex]

The two exponents can multiply together, so now it'll be [tex]8^{\frac{4}{12} }[/tex] which is [tex]8^{\frac{1}{3} }[/tex] when reduced

[tex]8^{\frac{1}{3} }[/tex] is the same thing as [tex]\sqrt[3]{8}[/tex]

Now, think about which number, when multiplied by itself 3 times is equal to 8... that number is 2--your answer

Answer:

2

Step-by-step explanation:

make it simple.. use a scientific calculator.

= ¹²√8⁴

= ¹²√4096

= 2

help me with it please with steps

class 9 trigonometry ​

Answers

secA=√2

secA=sec45

A=45°

3cos²45+5tan²45/ 4tan²A-sin²45

= 3(1/√3)²+5(1)² / 4(1)²-(1/√2)²

=3/2+5/4-1/2

=13/7

Answer:

[tex]\frac{13}{7}[/tex]

Step-by-step explanation:

Using the identities

cos x =[tex]\frac{1}{secx}[/tex] , sin²x = 1 - cos²x

tan²x = sec²x - 1

Given

[tex]\frac{3cos^2A+5tan^2A}{4tan^2A-sin^2A}[/tex]

= [tex]\frac{3(\frac{1}{sec^2A}+5(sec^2A-1) }{4(sec^2A -(1-cos^2A)}[/tex]

= [tex]\frac{3.(\frac{1}{\sqrt{2})^2 } +5((\sqrt{2})^2-1) }{4((\sqrt{2})^2-1)-(1-(\frac{1}{\sqrt{2} })^2 }[/tex]

= [tex]\frac{\frac{3}{2}+5(2-1) }{4(2-1)-(1-\frac{1}{2} )}[/tex]

= [tex]\frac{\frac{3}{2} +5}{4-\frac{1}{2} }[/tex]

= [tex]\frac{\frac{13}{2} }{\frac{7}{2} }[/tex]

= [tex]\frac{13}{2}[/tex] × [tex]\frac{2}{7}[/tex]

= [tex]\frac{13}{7}[/tex]

Trigonometric ratios
class 9
please answer my questions​

Answers

Step-by-step explanation:

Hi there!

Please see the answer in the picture.

Hope it helps!

1. Approach

One is given a trigonometric equation with and one is asked to prove that it is true. Using the attached image, combined with the knowledge of trigonometry, one can evaluate each trigonometric function. Then one can simplify each ratio to solve. To yield the most accurate result, one has to each of the ratios in a fractional form, rather than simplifying it into a decimal form. Remember the right angle trigonometric ratios, these ratios describe the relationship between the sides and angles in a right triangle. Such ratios are as follows,

[tex]sin(\theta)=\frac{opposite}{hypotenuse}\\\\cos(\theta)=\frac{adjacent}{hypotenuse}\\\\tan(\theta)=\frac{opposite}{adjacent}\\\\csc(\theta)=\frac{hypotenuse}{opposite}\\\\sec(\theta)=\frac{hypotenuse}{adjacent}\\\\cot(\theta)=\frac{adjacent}{opposite}[/tex]

Please note that the terms (opposite) and (adjacent) are relative to the angle uses in the ratio, however the term (hypotenuse) refers to the side opposite the right angle, this side never changes its name. Use these ratios to evaluate the trigonometric functions. Then simplify to prove the identity.

2. Problem (9)

[tex]\frac{sin(60)+cos(30)}{1+sin(30)+cos(60)}=sin(60)[/tex]

As per the attached image, the following statements regarding the value of each ratio can be made:

[tex]sin(60)=\frac{\sqrt{3}}{2}\\\\cos(30)=\frac{\sqrt{3}}{2}\\\\sin(30)=\frac{1}{2}\\\\cos(60)=\frac{1}{2}[/tex]

Substitute,

[tex]\frac{sin(60)+cos(30)}{1+sin(30)+cos(60)}=sin(60)[/tex]

[tex]\frac{\frac{\sqrt{3}}{2}+\frac{\sqrt{3}}{2}}{1+\frac{1}{2}+\frac{1}{2}}=\frac{\sqrt{3}}{2}[/tex]

Simplify,

[tex]\frac{\frac{\sqrt{3}}{2}+\frac{\sqrt{3}}{2}}{1+\frac{1}{2}+\frac{1}{2}}=\frac{\sqrt{3}}{2}[/tex]

[tex]\frac{\frac{2\sqrt{3}}{2}}{1+\frac{1}{2}+\frac{1}{2}}=\frac{\sqrt{3}}{2}[/tex]

[tex]\frac{\frac{2\sqrt{3}}{2}}{1+1}=\frac{\sqrt{3}}{2}[/tex]

[tex]\frac{\sqrt{3}}{1+1}=\frac{\sqrt{3}}{2}[/tex]

[tex]\frac{\sqrt{3}}{2}=\frac{\sqrt{3}}{2}[/tex]

Thus, this equation is true.

2. Problem (10)

Use a similar strategy to evaluate this equation,

[tex]\frac{1-cos(30)}{sin(30)}=\frac{1-cot(60)}{1+cot(60)}[/tex]

Use the attached image to evaluate the ratios.

[tex]cos(30)=\frac{\sqrt{3}}{2}\\\\sin(30)=\frac{1}{2}\\\\cot(60)=\frac{1}{\sqrt{3}}[/tex]

Substitute,

[tex]\frac{1-cos(30)}{sin(30)}=\frac{1-cot(60)}{1+cot(60)}[/tex]

[tex]\frac{1-\frac{\sqrt{3}}{2}}{\frac{1}{2}}=\frac{1-\frac{1}{\sqrt{3}}}{1+\frac{1}{\sqrt{3}}}[/tex]

Simplify,

[tex]\frac{1-\frac{\sqrt{3}}{2}}{\frac{1}{2}}=\frac{1-\frac{1}{\sqrt{3}}}{1+\frac{1}{\sqrt{3}}}[/tex]

[tex]\frac{\frac{2-\sqrt{3}}{2}}{\frac{1}{2}}=\frac{1-\frac{1}{\sqrt{3}}}{1+\frac{1}{\sqrt{3}}}[/tex]

[tex]\frac{\frac{2-\sqrt{3}}{2}}{\frac{1}{2}}=\frac{\frac{\sqrt{3}-1}{\sqrt{3}}}{1+\frac{1}{\sqrt{3}}}[/tex]

[tex]\frac{\frac{2-\sqrt{3}}{2}}{\frac{1}{2}}=\frac{\frac{\sqrt{3}-1}{\sqrt{3}}}{\frac{\sqrt{3}+1}{\sqrt{3}}}[/tex]

[tex]2-\sqrt{3}=\frac{\frac{\sqrt{3}-1}{\sqrt{3}}}{\frac{\sqrt{3}+1}{\sqrt{3}}}[/tex]

[tex]2-\sqrt{3}=\frac{\sqrt{3}-1}{\sqrt{3}}*\frac{\sqrt{3}}{\sqrt{3}+1}}[/tex]

[tex]2-\sqrt{3}=\frac{\sqrt{3}-1}{\sqrt{3}+1}[/tex]

Rationalize the denominator,

[tex]2-\sqrt{3}=\frac{\sqrt{3}-1}{\sqrt{3}+1}[/tex]

[tex]2-\sqrt{3}=\frac{\sqrt{3}-1}{\sqrt{3}+1}*\frac{\sqrt{3}-1}{\sqrt{3}-1}[/tex]

[tex]2-\sqrt{3}=\frac{(\sqrt{3}-1)^2}{3-1}[/tex]

[tex]2-\sqrt{3}=\frac{3-2\sqrt{3}+1}{2}[/tex]

[tex]2-\sqrt{3}=\frac{4-2\sqrt{3}}{2}[/tex]

[tex]2-\sqrt{3}=2-\sqrt{3}[/tex]

Therefore, this equation is also true.

a number y cubed plus x squared decreased by 7

Answers

Step-by-step explanation:

It is read thus Y^3+X^2-7

The equation form from the given statement would be Y^3 + X^2 - 7.

How to form mathematical expression from the given description?

You can represent the unknown amounts by the use of variables. Follow whatever the description is and convert it one by one mathematically.

For example if it is asked to increase some item by 4 , then you can add 4 in that item to increase it by 4.

We have been given a statement as;

"A number y cubed plus x squared decreased by 7".

So, the equation form would be

Y^3 + X^2 - 7

Learn more about expression here;

https://brainly.com/question/14083225

#SPJ2

The complete question is

"A number y cubed plus x squared decreased by 7". write the expression.

Find tan a Picture included

Answers

Answer:

5/12

Step-by-step explanation:

We know its a right angled triangle because of phythogorean theorem,

h^2=p^2+b^2

Take angle A as reference,

13^2=12^2+5^2

or,169=144+25

169=169

Now,

We know,

Tanx=p/b

As we took angle A as reference,

p=5

b=12

h=13

Put that in tanx=p/b

We get,

tanx=p/b=5/12

Answer:

Step-by-step explanation:

[tex]tan \ A = \frac{opposite \ side}{adjacent \ side}\\\\tan \ A = \frac{5}{12}[/tex]

Find the approximate volume of this prism (Image down below)

Answers

Answer:

about 62m^3

Step-by-step explanation:

Is 6272 a perfect square? If not find the smallest number by which it must be divided so that the product becomes a perfect square.

Answers

Answer:

(2^7)*(7^2)

Step-by-step explanation:

2*2*2*2*2*2*2*7*7=6272where2 is the smallest number by which it becomes a perfect square

(algebra) grade 9

Do the ratios 2/1 and 4/8 form a proportion?



answers:
~yes~
or
~no~

Answers

no because 2/1 is two wholes while 4/8 is half a whole

Answer:

No

Step-by-step explanation:

Find a number you can multiply 2 by to get 4.

2

Okay, now to ensure the ratio stays the same, multiply by 2/2 (aka 1)

You'll get 4/2, 2hixh is NOT equal to 4/8. So no, these ratios aren't equal.

Hope this helps!

all integers are whole numbers?true or false​

Answers

Answer:

that would be false

Step-by-step explanation:

all whole numbers are integers, but not all integers are whole numbers

Match the vocab word

Answers

Answer:

1). Algebraic expression - a letter or symbol used to represent an unknown.

2). Coefficient - a numerical value.

3). Constant - the constant preceding the variables in a product.

4). Expression - a mathematical expression containing one or more variables.

5). Variable - a mathematical phrase that cannot be determined true or false.

when would you write an x in an equation?)

Answers

Answer:

[tex]\Large \boxed{\mathrm{When \ a \ number \ is \ not \ known}}[/tex]

Step-by-step explanation:

For example, a sum of a number and 6 is 12.

The number is unknown.

Let the number be x.

x + 6 = 12

We can solve for x (unknown number). Subtract 6 from both sides of the equation.

x = 6

Simply. If the solution is not a real number enter not a real number rotate picture answer all 3 please

Answers

Answer:

13. [tex]\frac{\sqrt[5]{x^4} }{x}[/tex].

14. [tex]v = \pm3\sqrt{5}[/tex]

15. 2.

Step-by-step explanation:

13. [tex]x^{1/5} * x^{-2/5}[/tex]

= [tex]x^{1/5 + (-2/5)}[/tex]

= [tex]x^{1/5 - 2/5}[/tex]

= [tex]x^{-1/5}[/tex]

= [tex]\frac{1}{x^{1/5}}[/tex]

= [tex]\frac{x^{4/5}}{x^{1/5 + 4/5}}[/tex]

= [tex]\frac{x^{4/5}}{x}[/tex]

= [tex]\frac{\sqrt[5]{x^4} }{x}[/tex].

14. [tex]v^2 - 45 = 0[/tex]

[tex]v^2 = 45[/tex]

[tex]\sqrt{v^2} = \pm\sqrt{45}[/tex]

[tex]\sqrt{v^2} = \pm\sqrt{3^2 * 5}[/tex]

[tex]v = \pm3\sqrt{5}[/tex].

15. [tex]\sqrt[3]{2} * \sqrt[3]{4}[/tex]

= [tex]\sqrt[3]{2 * 4}[/tex]

= [tex]\sqrt[3]{2 * 2 * 2}[/tex]

= [tex]\sqrt[3]{2 ^3}[/tex]

= 2.

Hope this helps!

I need urgent help with the question. Can someone please give full working out of the attached question

Answers

Answer:

2x-2

Step-by-step explanation:

Being specifically asked to rationalise the denominator we focus on it as required.

To rationalise a term it has to be multiplied by its inverse.

The inverse of

[tex] \sqrt{x} - \sqrt{2 - x} [/tex]

is

[tex] \sqrt{x} + \sqrt{2 - x} [/tex]

Multiplying this two vives us a difference of two squares

[tex] {( \sqrt{x} })^{2} - {( \sqrt{2 - x} )}^{2} [/tex]

which gives us

x-(2-x)

=2x-2

What characteristics do similar triangles share? a They have the same sides and angles. b They have the same sides but different angles. c They have the same ratios for the sides. d They are the exact same.

Answers

Answer:

b. they have the same sides but different angles

Step-by-step explanation:

this answer makes the most sense

Answer:

C they have the same ratios for the sides

Step-by-step explanation:

Two triangles are said to be similar if their corresponding angles are congruent and the corresponding sides are in proportion. In other words, similar triangles are the same shape, but not necessarily the same size. The triangles are congruent if, in addition to this, their corresponding sides are of equal length.

please answer if you can

Answers

Step-by-step explanation:

denominator

add numerator

the difference of two numbers is 24.if the smaller number is 38,find the greater one​

Answers

Answer:

62

Step-by-step explanation:

add 24 to the smaller number to get the greater one

Answer:

62

Step-by-step explanation:

let, the greater number be 'x' and smaller number be 'y'

x-y=24 i.e. x-38=24

or, X=24+38

X= 62

Find the missing value can someone help me get this

Answers

Answer:

-3

Step-by-step explanation:

-5 = -8 - (- 3)

the answer is -3 lololol

pls help with this question

Answers

Try this

(x^2 + x - 12) + (x^2 - 8x + 16)

2y - 1

√36

i wanted to help you! :)

Answer:

[tex]\frac{3}{3}[/tex] = 1   (everything basically cancels)

Step-by-step explanation:

find the lower quartile for the data {47.2, 33.8, 43, 62, 5.8, 9, 61.4, 30.8, 68.2, 51.6, 13.2, 17.4, 64.2, 50.6, 29.4, 40.4}

Answers

Answer:

The lower quartile is 23.4

Step-by-step explanation:

The given data are;

47.2, 33.8, 43, 62, 5.8, 9, 61.4, 30.8, 68.2, 51.6, 13.2, 17.4, 64.2, 50.6, 29.4, 40.4

Rearranging the data, we have;

5.8, 9, 13.2, 17.4, 29.4, 30.8, 33.8, 40.4, 43, 47.2, 50.6, 51.6, 61.4, 62, 64.2, 68.2

The lower quartile, Q₁, is the (n + 1)/4 th term which is (16 +1)/4 = 4.25th term

However since we have an even set of numbers, we place a separator at the middle and we look for the median of the left half as follows

5.8, 9, 13.2, 17.4, 29.4, 30.8, 33.8, 40.4║ 43, 47.2, 50.6, 51.6, 61.4, 62, 64.2, 68.2

We have two numbers (17.4 + 29.4) at the median of the left set of numbers, we find the average of the two numbers to get the lower quartile

The lower quartile is therefore = (17.4 + 29.4)/2 = 23.4.

The common difference of an ap is -2 find its sum of first term is hundred and last term is -10 with full solution

Answers

Answer:

2520

Step-by-step explanation:

The n th term of an AP is

[tex]a_{n}[/tex] = a₁ + (n - 1)d

where a₁ is the first term and d the common difference.

To find the number of terms given [tex]a_{n}[/tex] = - 10 , d = - 2 and a₁ = 100, then

100 - 2(n - 1) = - 10 ( subtract 100 from both sides )

- 2(n - 1) = - 110 ( divide both sides by - 2 )

n - 1 = 55 ( add 1 to both sides )

n = 56 ← number of terms

Given n, a₁ and a₅₆ , then the sun of the terms is

[tex]S_{56}[/tex] = [tex]\frac{n}{2}[/tex] (a₁ + a₅₆ )

      = [tex]\frac{56}{2}[/tex] (100 - 10) = 28 × 90 = 2520

after allowing 20% discount an article is sold for rs.672 levying 12% VAT, find its market price​

Answers

The market price is Rs. 750 which was obtained by creating a mathematical relationship from the given parameters.

PERCENTAGE DISCOUNT = 20%

VAT LEVIED= 12%

PRICE SOLD = 672

Let the MARKET PRICE = m

Hence,

market price * (1 - discount) * (1 + VAT) = price sold

m * (1 - 20%) * (1 + 12%) = 672

m * (1 - 0.2) * (1 + 0.12) = 672

m * 0.8 * 1.12 = 672

0.896m = 672

m = 672 / 0.896

m = Rs. 750

Learn more :

https://brainly.com/question/20418815

The Market Price of the product is RS. 750.

The Market Price is calculated by dividing the components associated to Discount, which is less than 1, and the Value Added Tax, which more than 1, to the Resulting Price.

[tex]c_{M} = \frac{c_{R}}{\left(1-\frac{r_{D}}{100} \right)\cdot \left(1+\frac{r_{T}}{100} \right)}[/tex] (1)

Where:

[tex]c_{M}[/tex] - Market price, in monetary units.

[tex]c_{R}[/tex] - Resulting price, in monetary units.

[tex]r_{D}[/tex] - Discount rate, in percentage.

[tex]r_{T}[/tex] - Tax rate, in percentage.

If we know that [tex]c_{R} = 672[/tex], [tex]r_{D} = 20[/tex] and [tex]r_{T} = 12[/tex], then the market price is:

[tex]c_{M} = \frac{672}{\left(1-\frac{20}{100} \right)\cdot \left(1+\frac{12}{100} \right)}[/tex]

[tex]c_{M} = 750[/tex]

The market price of the product is RS. 750.

Three identical squares are placed side by side to form a rectangle with a perimeter of 104 inches. What is the area, in square inches, of each square?​

Answers

Answer:

13 inches

Step-by-step explanation:

I have done a Little draw because I don't know how i can explain you this exuse me

what is the x intercept for y=(2x(x-3))/(x-1). ?

my answer sheet says it is supposed to be -0.5​

Answers

Answer:

( 0,0) and (3,0)

Step-by-step explanation:

y=(2x(x-3))/(x-1)

Set y = 0

0 =(2x(x-3))/(x-1)

Multiply each side by x-1, which makes x not equal to 1

0 =(2x(x-3))

Using the zero product property

2x=0  x-3 =0

x=0   x= 3

The x intercepts are x=0   x= 3

( 0,0) and (3,0)

Problem at the River-A girl with a fox ,a goose and a bag of corn wanted to cross a river she could only take one animal or object at a time the fox first,the goose if they were left alone together.if the girl took the fox first the goose would eat the corn . How did the girl get all there safely across the river?

Answers

This is a great riddle to get people thinking, because it's not an obvious answer!

River A, you have: a fox, a goose, and a bag of corn.

So, you've probably gone through this looking for some way to bring one of them over, but if she takes the cornmeal, the fox will eat the goose, if she takes the fox, the goose will eat the corn, and if she takes the goose, then the fox and the cornmeal will be left together (which isn't bad until you have to pick one of them to take over with the goose).

Let's start by picking one of the animals (or objects) to take over first. Because it takes two trips for things to get bad with the goose, will do it first.

We take the goose across river A, and come back to the fox and the cornmeal. Nothing bad has happened. Now we know that if we leave the goose with the fox or the cornmeal, the cornmeal or the goose will be eaten.

There is one other thing we can do! The girl can only take one thing back and forth, but she can bring something back when she takes one of them across!

This means that:

She can take the goose first and the cornmeal second, but when she takes the cormeal, she can grab the goose and bring it back with her, so now the cornmeal is across river A but the goose isn't! Then, she can drop off the gosse and grab the fox, because the fox won't do anything to the cornmeal, and then come back for the goose!

Just ask if you have an questions! :)

What is [tex]3^2*3^5[/tex]?

Answers

Answer:

[tex]3^7[/tex]

Step-by-step explanation:

[tex]3^2*3^5[/tex]

[tex]\text {Apply Product Rule: } a^b+a^c=a^{b+c}\\\\3^2*3^5=3^{2+5}=3^7[/tex]

3^7 or 2187. When you have the same number with exponents, you add the exponents together to get your answer

Complete the sequence
8,27,64,125,.........
Just next letter

Answers

Answer:

216.

Step-by-step explanation:

These numbers are perfect cubes starting with 2^3.

2^3, 3^3,  4^3,  5^3 so the next one is 6^3, which is 216.

PLEASE HELP ME FAST...
I will mark you a BRAINLEST​

Answers

Answer: Hi!

Row A: 7, 11, 15, 19, 23

Row B: 51, 42, 33, 24, 15

Row C: 4, 8, 16, 32, 64

Row D: 64, 32, 16, 8, 4

(If you notice, Row C and Row D are just swapped in order!)

Hope this helps!

Hi there! Hopefully this helps!

--------------------------------------------------------------------------------------------------------------

A's rule, add 4: 7, 11, 15, 19, 23

B's rule, Subtract 9: 51, 42, 33, 24, 15

C's rule, Multiply by 2: 4, 8, 16, 32, 64

D's rule, Divide by 2: 64, 32, 16, 8, 4

Can someone please do this for me please

Answers

Answer:

r=-11

Step-by-step explanation:

7r+2=5(r-4)

7r+2=5r-20

2r=-22

r=-11

I hope it helps you!!have a good day

find the positive square root of 26.77

Answers

Answer:5.173973328

Step-by-step explanation:

√ 26.77 = 5.17
Thus: 5.1739733 x 5.1739733 = 26.77

T. For a science experiment, Katie had
to cool one sample bacteria to -52°C.
She had to cool a second sample
bacterium to -79°C. What is the
difference in the two temperatures?
Please help me figure out the equation too.

Answers

Answer:

Step-by-step explanation:

A "difference" indicates subtraction is being used. The difference between these 2 numbers is found the same way we find the distance between 2 points on a number line. Since both the difference and the distance have to be positive numbers, the formula to find it is

|a - b| which is the absolute value of a - b.

If a = -52 and b = -79:

| -52 - (-79) | = | -52 + 79 | = 27 OR

If a = -79 and b = -52:

| -79 - (-52) | = | -79 + 52 | = | -27 | = 27

Other Questions
When a schedule of reinforcement is variable, ___________. the rule about reinforcement stays the same the rule about reinforcement never changes the rule about reinforcement changes based on an average number there are no rules about reinforcement Which expression would be easier to simplify if you used the commutative property to change the order of the numbers? A 590-turn solenoid is 12 cm long. The current in it is 36 A . A straight wire cuts through the center of the solenoid, along a 4.5-cm diameter. This wire carries a 27-A current downward (and is connected by other wires that don't concern us).What is the magnitude of the force on this wire assuming the solenoid's field points due east? Simn Bolvar is best known for A. leading the fight for independence from Spain in South America. B. setting up dictatorships in South America following the collapse of Spanish rule. C. redistributing land and oil wealth among people in South America. D. launching paramilitary attacks against post-independence governments in South America. What is the built-in pollution control system in an incinerator called Dr. Edward is testing the same individuals repeatedly over time as part of a research study, while Dr. Janice is comparing the performance of different people of various ages at the same time. Dr. Edward is using a ________ research method; Dr. Janice is using a ________ method. The radius of a right circular cylinder is increasing at the rate of 7 in./sec, while the height is decreasing at the rate of 6 in./sec. At what rate is the volume of the cylinder changing when the radius is 20 in. and the height is 16 in. hlol can u plz solve b no plz plz.....I will mark as brainliest given a right-angled triangle with area 30 square centimeters and one of the legs of the right-angled triangle has 5cm, of the sum of the altitudes of the triangle can be expressed as a/b where gcd(a,b)=1. find a+b.geometry solve y+1/2-2y-1/2=4 words that can be formed from wager Net sales$688,500 $450,000 Cost of goods sold 337,364 133,200 Determine the 2016 and 2017 trend percents for net sales using 2016 as the base year. wo independent samples have been selected, 100 observations from population 1 and 76 observations from population 2. The sample means have been calculated to be x1=11.9 and x2=12.9. From previous experience with these populations, it is known that the variances are 21=27 and 22=23. (a) Determine the rejection region for the test of Kylie has a loyalty card good for a 13% discount at her local hardware store. What number should she multiply the prices on the tags by to find the price she would have to pay, before tax, in one step? What effect will replacing x with (x3) have on the graph of the equation y=x2 y = x 2 ? A. slides the graph 3 units down B. slides the graph 3 units right C. shrinks the graph by a factor of 3 D. slides the graph 3 units left Read the itinerary for a short trip to Louisiana and answer the questions in English.VENEZ EN LOUISIANE!SUGGESTIONS POUR UN COURT SJOURJOUR 1: Voyage en avion; arrive La Nouvelle-Orlans.JOUR 2: Faites un tour du Vieux Carr pied. Visitez le muse, le Presbytre, pour dcouvrir lhistoire de la ville. Dgustez des spcialits croles chez Antoine, le restaurant le plus vieux de la ville.JOUR 3: Partez tt en car pour la route des plantations. Dcouvrez des styles darchitecture diffrents: le style classique du sud amricain Oak Alley et le style crole Laura, par exemple. Passez la nuit la plantation Nottoway.JOUR 4: Allez Lafayette. En route, faites une balade en bateau sur le bassin Atchafalaya. Passez la nuit chez un habitant cadien. Dnez chez Prjean pour dcouvrir le jambalaya, le gombo et ltouffe. Allez danser au son de la musique cadienne chez Randols.JOUR 5: Allez au muse du parc national Jean Lafitte pour voir le film sur le Grand Drangement. Visitez le muse Vermillionville. Retournez La Nouvelle-Orlans.JOUR 6: Retour en France.1. What are the four different types of transportation mentioned?Enter Text2. What are two of the three cultures tourists will learn about?Enter Text3. What can one discover at the Presbytre?Enter Text4. What is one type of architecture seen in the plantations?Enter Text5. What are two Cajun specialties?Enter Text Consider the single factor APT. Portfolio A has a beta of 1.7 and an expected return of 19%. Portfolio B has a beta of .6 and an expected return of 15%. The risk-free rate of return is 11%. If you wanted to take advantage of an arbitrage opportunity, you should take a short position in portfolio __________ and a long position in portfolio _________. Multiple Choice A;A A;B B;B B;A Jerry deposited $10,000 in a bank account, and 10 years later he closes out the account, which is worth $18,000. The annual rate of interest that Jerry has earned over the 10 years is closest to: Use the formula for the volume of a cylinder to find the volume of a water tank (in ft with a radius of 15 feet and a height of 35 feet. Round to two decimal places. Suppose there are approximately 220 calories in 3 servings of a particular brand of cereal and 362 calories in 5 servings of the same cereal. Find the rate of change.